Difference between revisions of "2025 AMC 8 Problems/Problem 1"

(Undo revision 215691 by Theumbrellaacademy (talk))
(Tag: Undo)
(Undo revision 215682 by Theumbrellaacademy (talk))
(Tag: Undo)
 
(One intermediate revision by the same user not shown)
Line 1: Line 1:
Let m and n be 2 integers such that m > n. Suppose m + n = 20, m² + n² = 328, find m² - n².
+
The 2025 AMC 8 is not held yet. '''Please do not post false problems.'''
 
 
A) 280
 
B) 292
 
C) 300
 
D) 320
 
E) 340
 
 
 
<math>\textbf{(A)}\ 280 \qquad \textbf{(B)}\ 292 \qquad \textbf{(C)}\ 300 \qquad \textbf{(D)}\ 320 \qquad \textbf{(E)}\ 340</math>
 

Latest revision as of 08:08, 24 April 2024

The 2025 AMC 8 is not held yet. Please do not post false problems.